Does the Series Converge for Different Values of p and q?

In summary, the problem asks to find the values of real numbers p and q for which the series ## \sum \frac{1}{n^{p}ln^{q}(n)} ## either diverges or converges. The solution involves considering different cases for p and q, including values greater than and less than 1. By using the comparison test and the integral test, it is determined that the series converges for p,q > 1 and diverges for p,q ≤ 1. The boundary value of p,q=1 is also investigated.
  • #1
DotKite
81
1

Homework Statement



Show for what real numbers p and q ## \sum \frac{1}{n^{p}ln^{q}(n)} ## diverges or converges.

Homework Equations





The Attempt at a Solution



I am kind of lost because it seems that with both subscripts p and q there are a bunch of cases you have to work through. My professor wasn't explicit in regards to what values p and q we had to consider.

My number 1 question would be: do you consider p and q covering the same set of numbers? Such as p>1 and q>1? Or do you have to also consider 0<p<1 while q>1 as well? It just seems this would take forever.

Anyways I think I have a solution for p,q > 1


## \frac{1}{n^{p}ln^{q}(n)} ## ≤ ## \frac{1}{n^{p}} ## for all n ≥ 3

since ## \frac{1}{n^{p}} ## converges (p > 1) by comparison test ## \sum \frac{1}{n^{p}ln^{q}(n)} ## converges for p,q > 1
 
Physics news on Phys.org
  • #2
You can extend your comparison test more, what if q=0 and p>1 for example?

In general you are going to have to consider p and q being any number - there aren't that many actual different cases though
 
  • #3
## \frac{1}{n^{p}ln^{q}(n)} \geq \frac{1}{nln(n)} ## for 0 ≤ p,q ≤ 1.

and ## \frac{1}{nln(n)} ## is divergent by the integral test, thus ## \frac{1}{n^{p}ln^{q}(n)} ## diverges by comparison.

Now if p,q < 0 then ## \sum \frac{1}{n^{p}ln^{q}(n)} ## becomes ## \sum n^{p}ln^{q}(n) ##

which clearly diverges.

Ok, so all of this together we have the series converging for p,q > 1 and diverging for p,q ≤ 1. So are the other cases I need to check are ones with p and q being in these two different ranges respectively? ie p ≤ 1 and q > 1 and vice versa?
 
Last edited:
  • #4
You saw that p,q=1 is a sort of boundary value. Then it's worthwhile to investigate it deeper.

You could check what happens if you hold still p=1 and change q, then q=1 and vary p.
 

1. What is the definition of series converging when p and q are both greater than 1?

Series converging is a mathematical concept that describes the behavior of a sequence of numbers when p and q are both greater than 1. It means that as the number of terms in the sequence increases, the sum of the terms will approach a finite value.

2. How do you know if a series converges when p and q are both greater than 1?

To determine if a series converges when p and q are both greater than 1, you can use the ratio test or the root test. If the limit of the ratio or root of the terms is less than 1, then the series converges. If the limit is greater than 1, then the series diverges.

3. Can a series converge if p and q are both greater than 1?

Yes, a series can converge when p and q are both greater than 1. This happens when the terms of the series decrease fast enough to offset the effect of p and q being greater than 1. This can be determined using the ratio or root test as mentioned in the previous answer.

4. What happens if p or q is less than or equal to 1?

If p or q is less than or equal to 1, then the series will not converge. In fact, if p or q is less than or equal to 0, the series will diverge. This is because the terms of the series will not decrease fast enough to offset the effect of p and q being less than or equal to 1.

5. Are there any real-life applications of series converging when p and q are both greater than 1?

Yes, there are many real-life applications of series converging when p and q are both greater than 1. One example is in finance, where the concept is used to calculate compound interest on investments. It is also used in physics and engineering to model and predict the behavior of dynamic systems.

Similar threads

  • Calculus and Beyond Homework Help
Replies
2
Views
187
  • Calculus and Beyond Homework Help
Replies
3
Views
550
  • Calculus and Beyond Homework Help
Replies
1
Views
258
  • Calculus and Beyond Homework Help
Replies
3
Views
791
  • Calculus and Beyond Homework Help
Replies
3
Views
416
  • Calculus and Beyond Homework Help
Replies
2
Views
737
  • Calculus and Beyond Homework Help
Replies
2
Views
711
  • Calculus and Beyond Homework Help
Replies
4
Views
308
  • Calculus and Beyond Homework Help
Replies
1
Views
782
Replies
8
Views
1K
Back
Top